Can a Matrix A² ever equal -I₃ in M₃(ℝ)?

Click For Summary
No matrix A in M₃(ℝ) can satisfy the equation A² = -I₃. The discussion revolves around proving this statement using properties of determinants. It is suggested to compute the determinant of A to aid in the proof. The participants express gratitude for the assistance received in understanding the problem. The conclusion reinforces the impossibility of such a matrix existing.
AllRelative
Messages
42
Reaction score
2

Homework Statement


Show that no matrix A ∈ M3 (ℝ) exists so that A2 = -I3

Homework Equations

The Attempt at a Solution


This is from a french textbook of first year linear algebra. I'm quite familiar with properties of matrices but I don't have any idea of how to prove this.

Thanks for the help!
 
Physics news on Phys.org
Think about the determinants in the given equation.
 
Compute the determinant of ##A##.
 
I took me a while but I got it. Thanks both of you!
 
Question: A clock's minute hand has length 4 and its hour hand has length 3. What is the distance between the tips at the moment when it is increasing most rapidly?(Putnam Exam Question) Answer: Making assumption that both the hands moves at constant angular velocities, the answer is ## \sqrt{7} .## But don't you think this assumption is somewhat doubtful and wrong?

Similar threads

  • · Replies 2 ·
Replies
2
Views
3K
  • · Replies 1 ·
Replies
1
Views
2K
  • · Replies 5 ·
Replies
5
Views
5K
  • · Replies 2 ·
Replies
2
Views
4K
Replies
8
Views
3K
  • · Replies 8 ·
Replies
8
Views
2K
  • · Replies 1 ·
Replies
1
Views
2K
  • · Replies 7 ·
Replies
7
Views
4K
  • · Replies 2 ·
Replies
2
Views
2K
  • · Replies 5 ·
Replies
5
Views
3K